LSAT and Law School Admissions Forum

Get expert LSAT preparation and law school admissions advice from PowerScore Test Preparation.

 Administrator
PowerScore Staff
  • PowerScore Staff
  • Posts: 8917
  • Joined: Feb 02, 2011
|
#23389
Complete Question Explanation

Parallel Flaw-SN. The correct answer choice is (B)

This Parallel question is tricky because it seems at first to be logical. Authors on the LSAT often trick test-takers by using words like "many" or "most" about two different groups but implying that they are the same. In this stimulus, eligible students are those that have taken a course and have an interest. The second premise is that many students have an interest but have not taken a course. The conclusion is then that many student who want to participate will be ineligible. The first many deals with the all the students that have an interest in archaeology, but the second deals only with those who want to participate in the dig. We cannot assume that every student who has an interest in the field wants to participate in the dig. Also, notice the unique conclusion to this argument: it basically says, "many are not (eligible)."

Answer choice (A): This conclusion says basically that "there are some that are not," which is a different conclusion than "many are not." Also, the reasoning in this choice is valid.

Answer choice (B) This is the correct answer choice, we see the same beginning as in the stimulus: there are two conditions set for a well-schooled horse to be ideal, surefooted and gentle. We are then told that many surefooted horses are not gentle; the "many" deals with all surefooted horses. The conclusion, that many well-schooled horses are not ideal, is talking about many well-schooled horses, a different population than all surefooted horses. Like the stimulus, this answer confuses two populations. It could be that many surefooted horses are not gentle, but that most well-schooled and surefooted horses are, which would make the conclusion false. This is the same type of flawed reasoning, and you can immediately zoom in on this answer by noticing that the conclusion doubles perfectly: "many are not."

Answer choice (C): The conclusion of this answer choice is "not many," which is much different than "many are not." Also, if you were to diagram this answer choice, you would find it is a Mistaken Negation, which is not the flaw we are dealing with in the stimulus. Both of the "manys" in this argument deal with "first novels."

Answer choice (D): This answer choice has a conclusion of "many are," which is the opposite of the conclusion in the stimulus: "many are not." Also, this argument is really nonsensical if you think about it. Old cars need repairs if you don't maintain them. There are a lot of new cars that do not need repairs. Therefore, many automobiles are regularly maintained. Both "manys" in this argument deal with automobiles, and, although it is an obviously flawed method of reasoning, it does not match the stimulus.

Answer choice (E): The conclusion here "few will be" is really the same as the conclusion in answer choice C "not many become." but logically different from the conclusion in the stimulus of "many are not." This reasoning is flawed because it fails to consider the other possibility for being a good investment: providing a lot of office space.
 reop6780
  • Posts: 265
  • Joined: Jul 27, 2013
|
#12741
I got this question right, but couldn't figure out the flaw yet.

The answer was obviously answer B with almost identical structure.

Personally, "flaw" section is my weakness, and was wondering what flaw would be identified in this stimuli.

Thank you.
 Ron Gore
PowerScore Staff
  • PowerScore Staff
  • Posts: 220
  • Joined: May 15, 2013
|
#12805
Hi, Reop.

The stimulus is flawed because it assumes the students who have expressed an interest in archeology but have not taken an archaeology class want to participate in the dig. This assumption conflates two distinct conditions described in the first sentence of the stimulus: 1) student who wants to participate in a certain archeological dig (from the sufficient condition); and, 2) show an interest in archaeology (from the necessary condition). In other words, just because the student has shown an interest in archaeology generally does not mean they want to participate in this "certain" archaeological dig.

Hope that helps!

Ron
 Brandonhsi
  • Posts: 16
  • Joined: Jul 12, 2014
|
#18735
Hello,

I studied this question in the "Advanced LSAT Logical Reasoning Course," but I am still confused about what exact the flaw is (or how to clearly define it).

When I first read it, I thought the conclusion is just the contrapositive of the first part of stimulus. After reading it closely, I believe if the conclusion is "many student want to participate in the dig ALSO show an interest in the field will be ineligible to do so", it would make this conclusion valid?? Thanks!

Brandon
 Emily Haney-Caron
PowerScore Staff
  • PowerScore Staff
  • Posts: 577
  • Joined: Jan 12, 2012
|
#18737
Hi Brandon,

The flaw here is that we do not know that the students who show an interest in the field but have not taken a course WANT to participate in the dig; it is possible that all of those ineligible students actually don't even want to go. The same flaw appears in B: we do not know that the horses who are sure-footed but not gentle are well schooled horses. Without that information, the conclusion does not follow.
 kristinaroz93
  • Posts: 160
  • Joined: Jul 09, 2015
|
#19196
page 8-73 # 11 from the lesson 5-8 booklet: It starts of by saying, "At Flordyce University..."
I have diagrammed it as follows:
want to Participate in archeoloigcal dig-->arcehology course + show interest
not archeology course or not interest --> not dig
conclusion: many students who have shown an interest have never taken one archeology course--> many who want to cannot participate in the dig.
It does look like a valid argument from how I approached it. Then I came upon the explanation that the flaw is that two different populations are being referred to, "many" who have shown interest and "many" who want to particiapte in the dig. I cannot wrap my head aroud how these are two different populations, and that we cannot assume that every person who has shown an interest in the field wants to participate in the dig. Even the stimulus starts of by saying the necessary conditions required of students who WANT to participate in a certain archeological dig (e.g.show interest + course). To me this looks like a valid argument, so how else may I approach this problem? What am I missing here?
 Adam Tyson
PowerScore Staff
  • PowerScore Staff
  • Posts: 5153
  • Joined: Apr 14, 2011
|
#19199
Kristin,

I've taken the liberty of splitting your post in two, one for each question, and will tackle them both for you that way.

For this question, let's dig into the first sentence and make sure we have the right conditions. We might uncover something useful there.

You've identified the sufficient condition as wanting to participate, but is that really it? Read more closely and you may find that the real issue here is eligibility - anyone who wants to is eligible only if those two necessary conditions are met (taking a course and showing an interest). Your diagram might read Eligible -> Course & Interest.

We learn that there are many students interested in archaeology who haven't taken a course, and the contrapositive tells us that they are therefore not eligible. Does the conclusion stop there, saying that "many students are ineligible"? If it did, it would be valid, but it goes off the rails when the author assumes that many of those ineligible students actually want to participate in a dig. Being interested in the study of the subject doesn't automatically mean you want to take part in every aspect of it, right? I'm interested in the study of the law, but that doesn't mean I want to hang out in court all day! It could be that there are lots of students who have some level of interest in archaeology but who never want to get their hands dirty at a dig site. Maybe the only ones that want to go to an actual dig have all taken a course, and are therefore eligible?

When you compare that to Answer B, you'll find the same flaw - the author assumes, without justification, that there are many horses that are not gentle that are nevertheless well schooled. What if that isn't true? What if well schooled horses are ALL gentle, and the ones that are not gentle are not well schooled? It's that unsupported assumption that we are looking for in this parallel flaw question.

I hope that made it clearer!
 kristinaroz93
  • Posts: 160
  • Joined: Jul 09, 2015
|
#19202
While I do see that I missed the tibbit on eligibility in the suffificent condition, I still do not see why we ignore the term want in the first sentence. Shouldn't the sufficient be somewhere along the lines of: wants to participate and eligibe-->course + interest.

I don't know why I am struggling so much to wrap my head around this problem=/
 Adam Tyson
PowerScore Staff
  • PowerScore Staff
  • Posts: 5153
  • Joined: Apr 14, 2011
|
#19206
I see your point - so let's carry through on that. The contrapositive of your diagram is:

course or interest -> eligible or want

Does that make sense here? If it did, then there could be someone who didn't take a course who is nonetheless eligible, so long as they don't want to go (only one of those two conditions has to be met in this diagram for it to be valid). But we know that they simply aren't eligible if they don't meet both criteria, so the wanting is sort of irrelevant, isn't it?

Instead, think of the "wanting" as being a pre-qualifier of sorts - we are only interested in the group of people that want to go on a dig. It's sort of like a "people who" indicator - it's "wanters who are eligible have taken a course and shown an interest". When we have a "people who" argument, we generally don't include "people" as a sufficient condition, we just keep in mind that we are talking about people and not dogs or cars or mushrooms, and then we look at the next thing in the relationship - people who "X".

I hope that makes some sense! Sometimes we just have to play with things a bit to see how they fit together. I'm not sure I am doing such a hot job of explaining it here, but try playing with it yourself to find out what role "want" plays in the argument.
 medialaw111516
  • Posts: 80
  • Joined: Dec 11, 2018
|
#71677
Hi,

I'm a little confused as to how B can be correct over E. I looked at all of the things discussed in the setup, but I guess I was looking for the conditional "only if" language to be in the correct answer choice. Is the "must also" taking thr place of that in B or did I miss something more important?

Get the most out of your LSAT Prep Plus subscription.

Analyze and track your performance with our Testing and Analytics Package.